Tension for the crown

The blacksmiths of the kingdom of Mechania are wondering how much tension is holding a crown together as it rests on the head of their sovereign. The king of Mechania has a frictionless, perfectly spherical head with radius r = 10 cm r = 10 \text{ cm} , and the crown is a thin cord with a length of = 37.7 cm \ell = 37.7 \text{ cm} and a mass of m = 628.32 g . m = 628.32 \text{ g}.

Compute the tension in the crown (in Newtons).

Assumptions:

  • The crown has no thickness and rests perfectly level (that is, it makes a circle which is contained in a plane parallel to the horizontal).
  • The gravitational constant of Mechania is g = 10 m/s 2 . g = 10 \text{ m/s}^2.


The answer is 0.750029363.

This section requires Javascript.
You are seeing this because something didn't load right. We suggest you, (a) try refreshing the page, (b) enabling javascript if it is disabled on your browser and, finally, (c) loading the non-javascript version of this page . We're sorry about the hassle.

3 solutions

Antoine G
Jun 5, 2018

Here is my "energy-based solution" (see Steven Chase's post for a "force-based" solution).

The potential energy of the wire is given by E = m g h E = mgh (where h h is the height at which the wire lies; here I set the height to be 0 at the center of the sphere). This may be re-expressed as a function of the length \ell of the wire. Use h 2 + ( 2 π ) 2 = r 2 h^2 + (\frac{\ell}{2 \pi})^2 = r^2 to get: E = m g r 2 ( 2 π ) 2 E = mg\sqrt{ r^2 - (\tfrac{\ell}{2 \pi})^2 } Now imagine the wire would increase its length by d \text{d}\ell , then the energy would change as d E = F d \text{d}E = -F \text{d}\ell where F F is the tension (the change in energy corresponds to a negative work by the tension in the wire). Hence F = d E d = m g 1 r 2 ( 2 π ) 2 4 π 2 = m g 4 π 2 r 2 ( 2 π ) 2 \begin{array}{rl} F =& -\dfrac{ \text{d}E }{ \text{d}\ell }\\ = & mg \dfrac{1}{ \sqrt{ r^2 -(\tfrac{\ell}{2 \pi})^2 } } \dfrac{\ell}{4 \pi^2}\\ =& \dfrac{mg \ell}{4\pi^2 \sqrt{ r^2 -(\tfrac{\ell}{2 \pi})^2 } } \end{array} Pluging in the values give 0.75 N \approx \fbox{0.75 N} .

Please include all input needed, especially when you use g = 10 m/s² ...

mat baluch - 3 years ago

Log in to reply

oh sorry about that, I am sure I edited this in, but the edit was apperently not saved... Correcting immediately...

Antoine G - 3 years ago

"Uneasy lies the head that wears a crown" (2HIV Act III scene1 line 31) I should think so - nod the smallest amount and it'll fall off! Your method is similar to what in my day was called "virtual work" (an attractive prospect) - in an energy-preserving situation (no friction) stationary points of PE function give equilibrium. 2nd derivative determines whether equilibrium is stable or unstable. I solved this one simply by using resolution of forces (to establish that the total outward force from head to crown is 1.25mg at an angle of arcsine 0.6 to the vertical), and then triangle of forces with two T's, an angle of delta theta, and third side 1.25mg x 0.6 x delta theta/(2 pi), so T = 1.25 x 0.6 x 0.62832 x 10/(2 pi). I think that's much the same as Brian's solution.

A Former Brilliant Member - 2 years, 11 months ago

Log in to reply

nice quote ;o) (and nice solution)

It's a "trick" I learned from the Physics Olympiad back home... There was this other problem where one could try to determine the tension on soap bubbles. You had to start by blowing soap bubbles from a straw, then measure the time it takes for them to expunge all the air back (through the straw). Once you got that this time was proportional to r 4 r^4 (where r r is the radius of the soap bubble) you could then compute some forces/energy to get an idea of the tension. It was a cool problem, but I can't piece it together anymore.

Antoine G - 2 years, 11 months ago

good solution and awesome quote..XD

rajdeep brahma - 2 years, 11 months ago
Steven Chase
Jun 24, 2018

Suppose the ring is divided into many little pieces of angle d θ d \theta . Suppose also that each piece is acted on by a force d F = α d θ d F = \alpha \, d \theta , which points radially away from the sphere. The vectorial force components are:

d F x = α d θ c o s θ s i n ϕ d F y = α d θ s i n θ s i n ϕ d F z = α d θ c o s ϕ d F_x = \alpha \, d \theta \, cos \theta \, sin \phi \\ d F_y = \alpha \, d \theta \, sin \theta \, sin \phi \\ d F_z = \alpha \, d \theta \, cos \phi

The angle ϕ \phi is a fixed quantity determined by the parameters of the problem. We know that the integral of d F z d F_z must equal the weight.

F z = α c o s ϕ 0 2 π d θ = 2 π c o s ϕ α = m g α = m g 2 π c o s ϕ F_z = \alpha \, cos \phi \int_0^{2 \pi} d \theta = 2 \pi \, cos \phi \, \alpha = m g \\ \alpha = \frac{m g}{2 \pi \, cos \phi }

Calculate the x x force associated with an arbitrarily defined angle window θ f θ θ f -\theta_f \leq \theta \leq \theta_f :

F x = α s i n ϕ θ f θ f c o s θ d θ = m g t a n ϕ 2 π θ f θ f c o s θ d θ = m g t a n ϕ π s i n θ f F_x = \alpha \, sin \phi \int_{-\theta_f}^{\theta_f} cos \theta \, d \theta \\ = \frac{m g \, tan \phi}{2 \pi} \int_{-\theta_f}^{\theta_f} cos \theta \, d \theta = \frac{m g \, tan \phi}{\pi} \, sin \theta_f

We know that this x x force is countered by the twice the corresponding tension component, evaluated at θ f \theta_f . This is because the tension pulls on the partial ring segment equally at both ends.

m g t a n ϕ π s i n θ f = 2 T s i n θ f T = m g t a n ϕ 2 π 0.75 \frac{m g \, tan \phi}{\pi} \, sin \theta_f = 2 \, T \, sin \theta_f \\ T = \frac{m g \, tan \phi}{2 \pi} \approx 0.75

Great solution!!+1!!

rajdeep brahma - 2 years, 11 months ago

Log in to reply

Thanks.................

Steven Chase - 2 years, 11 months ago

I have a solution using forces but I don't know how to write like u guys do. Can any1 plzz teach me . I want to share my solution but I don't know how to.

Anuj Tripathi - 2 years, 11 months ago

Log in to reply

have u got the answer correct?here?

rajdeep brahma - 2 years, 11 months ago

You mean you don't know Latex? You can hover over other peoples' Latex to see how it's done

Steven Chase - 2 years, 11 months ago

I am unable to convince me how tension is the same throughout the string. It might seem silly to you but please answer me. If I assume that tension is the same I have a force based solution with me. Thanks in advance

Donepudi Aditya - 2 years, 11 months ago

Log in to reply

Any non-uniformity in the tension would violate symmetry. If you tried to describe the assumed variation in tension using an equation, you would have to choose a reference (start point). There is no reason to choose one reference over another.

Steven Chase - 2 years, 11 months ago
Brian Moehring
Jun 26, 2018

Here's a "force-based" solution using the non-standard analysis of infinitesimals. I don't mean this to be taken as a formal solution; rather, I show it to give insight into how some analysts think about this type of problem.

Warning: While the theorems of nonstandard analysis are consistent (under the same assumptions on the consistency of set theory in standard mathematics), your instructors will probably give you no points for using this method in a class for the standard methods.


Let O O denote the center of the spherical head, let C C denote the center of the circle formed by the crown, let Q Q denote an arbitrary point on the crown, and let T ( Q ) T(Q) denote the tension at Q Q . We note that the "weight density per radian" of the crown is m g 2 π \frac{mg}{2\pi} , the radius of the crown is C Q = 2 π CQ = \frac{\ell}{2\pi} , and the height of the crown above O O is O C = r 2 ( 2 π ) 2 . OC = \sqrt{r^2 - \left(\frac{\ell}{2\pi}\right)^2}.

Now let d θ d\theta denote an infinitesimal. We consider a section of the crown centered at Q Q and including d θ d\theta radians to either side. There is a tension force T ( Q ) T(Q) at each end of this section acting in the same direction as the ends. The net effect is a force acting in the direction of Q C \vec{QC} with magnitude 2 T ( Q ) sin ( d θ ) 2T(Q)\sin(d\theta) .

We now consider the forces acting on this small segment:

  • Its weight acts in the direction of C O \vec{CO} with magnitude m g 2 π 2 d θ = m g π d θ \frac{mg}{2\pi} \cdot 2\,d\theta = \frac{mg}{\pi}d\theta
  • The net tension acts in the direction of Q C \vec{QC} with magnitude 2 T ( Q ) sin ( d θ ) 2T(Q)\sin(d\theta) .
  • The normal force acts in the direction of O Q \vec{OQ} .

In order for these to balance, the sum of the weight and net tension vectors acting on the segment must act in the direction of Q O \vec{QO} , so by similar triangles, 2 T ( Q ) sin ( d θ ) m g π d θ = C Q O C = 2 π r 2 ( 2 π ) 2 \frac{2T(Q)\sin(d\theta)}{\frac{mg}{\pi}d\theta} = \frac{CQ}{OC} = \frac{\frac{\ell}{2\pi}}{\sqrt{r^2 - \left(\frac{\ell}{2\pi}\right)^2}} Here, we note that sin ( d θ ) d θ = 1 \frac{\sin(d\theta)}{d\theta} = 1 , so applying this and solving for the tension: T ( Q ) = m g 4 π 2 r 2 ( 2 π ) 2 T(Q) = \frac{mg\ell}{4\pi^2\sqrt{r^2 - \left(\frac{\ell}{2\pi}\right)^2}}

Using the given values yields T ( Q ) 0.75 N \boxed{T(Q) \approx 0.75 \mbox{ N}}

I want to make a few comments on my solution for those who may be incredulous about this method. By my count, I've only used nonstandard analysis to make three material claims in this solution. Here they are, in increasing importance:

  • The most controversial part might be seen as my writing sin ( d θ ) d θ = 1 \frac{\sin(d\theta)}{d\theta} = 1 , but this is actually equivalent to lim x 0 + sin x x = 1 \lim_{x\to 0^+}\frac{\sin x}{x} = 1 . That is, while it might seem to be controversial at first glance, it's actually the easiest to put back into the standard framework. It's also interesting, to me at least, that this is the only result from calculus we require---everything else is just geometry. (For those who know hyperreal analysis, yes, I know it should be st ( sin ( d θ ) d θ ) = 1 \text{st}\!\left(\frac{\sin(d\theta)}{d\theta}\right) = 1 , but I hope we can agree this is a harmless error in context)
  • I stated that the tension at each end of the infinitesimal segment around Q Q is equal to the tension at Q Q . This isn't assuming, as it appears to be, that the tension in the crown is constant. Rather, this is just a consequence of assuming the tension in the crown is continuous. This means that our final result can be seen as proving the tension is constant, under the assumption that the tension is continuous. In this case, of course, the fact that the tension is constant can be seen as a result of symmetry, so this is also fairly harmless. To be honest, my original write-up just used the fact that the tension is constant, but I generalized it slightly when I saw someone concerned about this assumption in another comment thread.
  • By far the most important place I used nonstandard analysis to make a claim is when I said the normal force acts in the direction of O Q \vec{OQ} . This can be justified in the standard framework as a limiting case, but it would have some notational baggage that makes it a little harder to talk about. This is actually the sole reason I decided to work in the infinitesimal framework.

To wrap up: Everything can be done in the standard framework by assuming d θ > 0 d\theta > 0 is a real number and then taking the limit d θ 0 + d\theta \to 0^+ , but the notation becomes a little more complicated. This is why in practice lots of analysts use non-standard analysis. It's almost always much easier to deal with and usually provides true statements when interpreted in the standard framework.

Brian Moehring - 2 years, 11 months ago

Log in to reply

nice solution! On the formal level, the solution (by that I mean the math and formulas) look almost the same as the "energy-based" solution I posted. Coincidence?

Antoine G - 2 years, 11 months ago

Log in to reply

Thanks.

Since both of us decided to write the answer in terms of the given quantities, it actually seems inevitable that our formulas should at least be very similar.

Actually, when I was writing my solution up, I saw that your solution had the same form of the formulas, and I thought about changing it just to be a little different, but I decided against it because the form itself seemed to give all the intermediate quantities approximately integer values. (e.g. I left l 2 π 6 cm \frac{l}{2\pi} \approx 6 \text{ cm} in the square root because it's simpler in computation)

I debated even writing the final formula as T ( Q ) = ( m g 2 π ) ( 2 π ) r 2 ( 2 π ) 2 T(Q) = \frac{\left(\frac{mg}{2\pi}\right)\left(\frac{\ell}{2\pi}\right)}{\sqrt{r^2 - \left(\frac{\ell}{2\pi}\right)^2}} to completely parallel my computation, but decided that looks a little cramped.

Brian Moehring - 2 years, 11 months ago

Log in to reply

@Brian Moehring yeah, I was a bit lazy in writing down the problem... I wanted the solution to be a relatively round number (3/4) but was too impatient to hide the cancellations with in a less obvious manner :)

Anyway, nonstandard analysis is simply a cool thing. I guess you could argue with your tutor if he decided to give less points for such a solution. Especially in a physics course!

Antoine G - 2 years, 11 months ago

0 pending reports

×

Problem Loading...

Note Loading...

Set Loading...